Difference between revisions of "2003 AIME I Problems/Problem 14"

(Problem)
(See also)
Line 5: Line 5:
  
 
== See also ==
 
== See also ==
 +
* [[2003 AIME I Problems/Problem 13 | Previous problem]]
 +
* [[2003 AIME I Problems/Problem 15 | Next problem]]
 
* [[2003 AIME I Problems]]
 
* [[2003 AIME I Problems]]

Revision as of 18:43, 4 November 2006

Problem

The decimal representation of $m/n,$ where $m$ and $n$ are relatively prime positive integers and $m < n,$ contains the digits 2, 5, and 1 consecutively, and in that order. Find the smallest value of $n$ for which this is possible.

Solution

See also